Category:Burali-Forti Paradox

From ProofWiki
Jump to navigation Jump to search

This category contains pages concerning Burali-Forti Paradox:


The ordinal of the set of all ordinals must be larger than any of the ordinals in that set.

Hence this would be an ordinal which is not contained in that set.

That is, it is an ordinal which is not contained in the set of all ordinals.

Hence the set of all ordinals is not an allowable set.

Pages in category "Burali-Forti Paradox"

The following 3 pages are in this category, out of 3 total.